Difference between revisions of "2010 AIME II Problems/Problem 9"
m (→Solution) |
(Semi-automated contest formatting - script by azjps) |
||
Line 1: | Line 1: | ||
− | == Problem | + | == Problem == |
− | Let <math>ABCDEF</math> be a regular hexagon. Let <math>G</math>, <math>H</math>, <math>I</math>, <math>J</math>, <math>K</math>, and <math>L</math> be the | + | Let <math>ABCDEF</math> be a [[regular polygon|regular]] [[hexagon]]. Let <math>G</math>, <math>H</math>, <math>I</math>, <math>J</math>, <math>K</math>, and <math>L</math> be the [[midpoint]]s of sides <math>AB</math>, <math>BC</math>, <math>CD</math>, <math>DE</math>, <math>EF</math>, and <math>AF</math>, respectively. The [[segment]]s <math>\overbar{AH}</math>, <math>\overbar{BI}</math>, <math>\overbar{CJ}</math>, <math>\overbar{DK}</math>, <math>\overbar{EL}</math>, and <math>\overbar{FG}</math> bound a smaller regular hexagon. Let the [[ratio]] of the area of the smaller hexagon to the area of <math>ABCDEF</math> be expressed as a fraction <math>\frac {m}{n}</math> where <math>m</math> and <math>n</math> are [[relatively prime]] positive integers. Find <math>m + n</math>. |
+ | __TOC__ | ||
==Solution== | ==Solution== | ||
− | |||
<center><asy> | <center><asy> | ||
defaultpen(0.8pt+fontsize(12pt)); | defaultpen(0.8pt+fontsize(12pt)); | ||
Line 39: | Line 39: | ||
− | label('A',A, | + | label('$A$',A,(1,0)); |
− | label('B',B,NE); | + | label('$B$',B,NE); |
− | label('C',C,NW); | + | label('$C$',C,NW); |
− | label('D',D, W); | + | label('$D$',D, W); |
− | label('E',E,SW); | + | label('$E$',E,SW); |
− | label('F',F,SE); | + | label('$F$',F,SE); |
− | label('G',G,NE); | + | label('$G$',G,NE); |
− | label('H',H, | + | label('$H$',H, (0,1)); |
− | label('I',I,NW); | + | label('$I$',I,NW); |
− | label('J',J,SW); | + | label('$J$',J,SW); |
− | label('K',K, S); | + | label('$K$',K, S); |
− | label('L',L,SE); | + | label('$L$',L,SE); |
− | label('M',M); | + | label('$M$',M); |
− | label('N',N); | + | label('$N$',N); |
− | label('O',(0,0)); | + | label('$O$',(0,0),NE); dot((0,0)); |
</asy></center> | </asy></center> | ||
Line 61: | Line 61: | ||
Let <math>O</math> be the center. | Let <math>O</math> be the center. | ||
− | |||
− | |||
===Solution 1=== | ===Solution 1=== | ||
+ | Let <math>BC=2</math> (without loss of generality). | ||
− | + | Note that <math>\angle BMH</math> is the vertical angle to an angle of regular hexagon, and so has degree <math>120^\circ</math>. | |
− | |||
− | Note that <math>\angle BMH</math> is the vertical angle to an angle of regular hexagon, | ||
Because <math>\triangle ABH</math> and <math>\triangle BCI</math> are rotational images of one another, we get that <math>\angle{MBH}=\angle{HAB}</math> and hence <math>\triangle ABH \sim \triangle BMH \sim \triangle BCI</math>. | Because <math>\triangle ABH</math> and <math>\triangle BCI</math> are rotational images of one another, we get that <math>\angle{MBH}=\angle{HAB}</math> and hence <math>\triangle ABH \sim \triangle BMH \sim \triangle BCI</math>. | ||
− | Using a | + | Using a similar argument, <math>NI=MH</math>, and |
− | |||
− | |||
− | |||
− | |||
− | |||
− | |||
− | < | + | <cmath>MN=BI-NI-BM=BI-(BM+MH).</cmath> |
− | <math> | + | Applying the [[Law of cosines]] on <math>\triangle BCI</math>, <math>BI=\sqrt{2^2+1^2-2(2)(1)(\cos(120^\circ))}=\sqrt{7}</math> |
− | < | + | <cmath>\begin{align*}\frac{BC+CI}{BI}&=\frac{3}{\sqrt{7}}=\frac{BM+MH}{BH} \\ |
+ | BM+MH&=\frac{3BH}{\sqrt{7}}=\frac{3}{\sqrt{7}} \\ | ||
+ | MN&=BI-(BM+MH)=\sqrt{7}-\frac{3}{\sqrt{7}}=\frac{4}{\sqrt{7}} \\ | ||
+ | \frac{\text{Area of smaller hexagon}}{\text{Area of bigger hexagon}}&=\left(\frac{MN}{BC}\right)^2=\left(\frac{2}{\sqrt{7}}\right)^2=\frac{4}{7}\end{align*}</cmath> | ||
− | Thus, answer is <math>\boxed{011}</math> | + | Thus, answer is <math>\boxed{011}</math>. |
===Solution 2=== | ===Solution 2=== | ||
− | + | We can use coordinates. Let <math>O</math> be at <math>(0,0)</math> with <math>A</math> at <math>(1,0)</math>, | |
− | |||
− | Let <math>O</math> be at <math>(0,0)</math> with <math>A</math> | ||
then <math>B</math> is at <math>(\cos(60^\circ),\sin(60^\circ))=\left(\frac{1}{2},\frac{\sqrt{3}}{2}\right)</math>, | then <math>B</math> is at <math>(\cos(60^\circ),\sin(60^\circ))=\left(\frac{1}{2},\frac{\sqrt{3}}{2}\right)</math>, | ||
Line 99: | Line 91: | ||
<math>D</math> is at <math>(\cos(180^\circ),\sin(180^\circ))=(-1,0)</math>, | <math>D</math> is at <math>(\cos(180^\circ),\sin(180^\circ))=(-1,0)</math>, | ||
− | < | + | <cmath>\begin{align*}&H=\frac{B+C}{2}=\left(0,\frac{\sqrt{3}}{2}\right) \\ |
− | + | &I=\frac{C+D}{2}=\left(-\frac{3}{4},\frac{\sqrt{3}}{4}\right)\end{align*}</cmath> | |
− | |||
<br/> | <br/> | ||
Line 117: | Line 108: | ||
Let's solve the system of equation to find <math>M</math> | Let's solve the system of equation to find <math>M</math> | ||
− | < | + | <cmath>\begin{align*}-\frac{\sqrt{3}}{2}(x-1)-\frac{3}{2}&=\frac{\sqrt{3}}{5}\left(x-\frac{1}{2}\right) \\ |
+ | -5\sqrt{3}x&=2\sqrt{3}x-\sqrt{3} \\ | ||
+ | x&=\frac{1}{7} \\ | ||
+ | y&=-\frac{\sqrt{3}}{2}(x-1)=\frac{3\sqrt{3}}{7}\end{align*}</cmath> | ||
− | + | Finally, | |
− | < | + | <cmath>\begin{align*}&\sqrt{x^2+y^2}=OM=\frac{1}{7}\sqrt{1^2+(3\sqrt{3})^2}=\frac{1}{7}\sqrt{28}=\frac{2}{\sqrt{7}} \\ |
+ | &\frac{\text{Area of smaller hexagon}}{\text{Area of bigger hexagon}}=\left(\frac{OM}{OA}\right)^2=\left(\frac{2}{\sqrt{7}}\right)^2=\frac{4}{7}\end{align*}</cmath> | ||
− | + | Thus, the answer is <math>\boxed{011}</math>. | |
− | |||
− | |||
− | |||
− | |||
− | |||
− | |||
− | |||
− | Thus, answer is <math>\boxed{011}</math> | ||
− | |||
− | |||
== See also == | == See also == | ||
{{AIME box|year=2010|num-b=8|num-a=10|n=II}} | {{AIME box|year=2010|num-b=8|num-a=10|n=II}} | ||
+ | |||
+ | [[Category:Intermediate Geometry Problems]] |
Revision as of 10:53, 6 April 2010
Problem
Let be a regular hexagon. Let , , , , , and be the midpoints of sides , , , , , and , respectively. The segments $\overbar{AH}$ (Error compiling LaTeX. Unknown error_msg), $\overbar{BI}$ (Error compiling LaTeX. Unknown error_msg), $\overbar{CJ}$ (Error compiling LaTeX. Unknown error_msg), $\overbar{DK}$ (Error compiling LaTeX. Unknown error_msg), $\overbar{EL}$ (Error compiling LaTeX. Unknown error_msg), and $\overbar{FG}$ (Error compiling LaTeX. Unknown error_msg) bound a smaller regular hexagon. Let the ratio of the area of the smaller hexagon to the area of be expressed as a fraction where and are relatively prime positive integers. Find .
Solution
Let be the intersection of and
and be the intersection of and .
Let be the center.
Solution 1
Let (without loss of generality).
Note that is the vertical angle to an angle of regular hexagon, and so has degree .
Because and are rotational images of one another, we get that and hence .
Using a similar argument, , and
Applying the Law of cosines on ,
Thus, answer is .
Solution 2
We can use coordinates. Let be at with at ,
then is at ,
is at ,
is at ,
Line has the slope of and the equation of
Line has the slope of and the equation
Let's solve the system of equation to find
Finally,
Thus, the answer is .
See also
2010 AIME II (Problems • Answer Key • Resources) | ||
Preceded by Problem 8 |
Followed by Problem 10 | |
1 • 2 • 3 • 4 • 5 • 6 • 7 • 8 • 9 • 10 • 11 • 12 • 13 • 14 • 15 | ||
All AIME Problems and Solutions |